matheraum.de
Raum für Mathematik
Offene Informations- und Nachhilfegemeinschaft

Für Schüler, Studenten, Lehrer, Mathematik-Interessierte.
Hallo Gast!einloggen | registrieren ]
Startseite · Forum · Wissen · Kurse · Mitglieder · Team · Impressum
Forenbaum
^ Forenbaum
Status Mathe
  Status Schulmathe
    Status Primarstufe
    Status Mathe Klassen 5-7
    Status Mathe Klassen 8-10
    Status Oberstufenmathe
    Status Mathe-Wettbewerbe
    Status Sonstiges
  Status Hochschulmathe
    Status Uni-Analysis
    Status Uni-Lin. Algebra
    Status Algebra+Zahlentheo.
    Status Diskrete Mathematik
    Status Fachdidaktik
    Status Finanz+Versicherung
    Status Logik+Mengenlehre
    Status Numerik
    Status Uni-Stochastik
    Status Topologie+Geometrie
    Status Uni-Sonstiges
  Status Mathe-Vorkurse
    Status Organisatorisches
    Status Schule
    Status Universität
  Status Mathe-Software
    Status Derive
    Status DynaGeo
    Status FunkyPlot
    Status GeoGebra
    Status LaTeX
    Status Maple
    Status MathCad
    Status Mathematica
    Status Matlab
    Status Maxima
    Status MuPad
    Status Taschenrechner

Gezeigt werden alle Foren bis zur Tiefe 2

Navigation
 Startseite...
 Neuerdings beta neu
 Forum...
 vorwissen...
 vorkurse...
 Werkzeuge...
 Nachhilfevermittlung beta...
 Online-Spiele beta
 Suchen
 Verein...
 Impressum
Das Projekt
Server und Internetanbindung werden durch Spenden finanziert.
Organisiert wird das Projekt von unserem Koordinatorenteam.
Hunderte Mitglieder helfen ehrenamtlich in unseren moderierten Foren.
Anbieter der Seite ist der gemeinnützige Verein "Vorhilfe.de e.V.".
Partnerseiten
Dt. Schulen im Ausland: Mathe-Seiten:Weitere Fächer:

Open Source FunktionenplotterFunkyPlot: Kostenloser und quelloffener Funktionenplotter für Linux und andere Betriebssysteme
StartseiteMatheForenDiskrete OptimierungWerkstatt LP
Foren für weitere Schulfächer findest Du auf www.vorhilfe.de z.B. Geschichte • Erdkunde • Sozialwissenschaften • Politik/Wirtschaft
Forum "Diskrete Optimierung" - Werkstatt LP
Werkstatt LP < Optimierung < Diskrete Mathematik < Hochschule < Mathe < Vorhilfe
Ansicht: [ geschachtelt ] | ^ Forum "Diskrete Optimierung"  | ^^ Alle Foren  | ^ Forenbaum  | Materialien

Werkstatt LP: LP aufstellen
Status: (Frage) beantwortet Status 
Datum: 18:49 Mi 13.05.2015
Autor: triad

Aufgabe
Eine Werkstatt hat eine Bohr- und 5 Fräsmaschinen, um Teile zu produzieren, die aus 2 Werkstücken bestehen. Die Werkstücke müssen jeweils gebohrt und gefräst werden. Die Bearbeitungszeiten der Werkstücke sind:

Werkstück   Bohren (minuten)   Fräsen (minuten)
1           3                  20
2           5                  15

Alle Maschinen sollen so ausgelastet sein, dass keine Maschine mehr als 30 Minuten pro Tag länger läuft als jede der anderen. Die Fräsmaschinen seien dabei alle (so) gleichmäig (wie möglich) ausgelastet. Ein Arbeitstag hat 8 Stunden.

Formulieren Sie das Problem, die Maschinen so auszulasten, dass die maximale Anzahl von kompletten
Teilen produziert wird, als LP (Lineares Programm).

Hallo Leute,

mein Problem bei dieser Aufgabe besteht schon darin, welche und wie viele Variablen für was einzusetzen sind. Brauche ich nun für jede Maschine eine Variable oder für jedes Werkstück eine oder beides?

Ich habe mal so angefangen: Die Summe der Bohr- und Fräsminuten überschreiten nicht 8 Stunden (480 Minuten), daher die zwei Nebenbedingungen [mm] (x_i [/mm] für Werkstück i)

$\ [mm] 3x_1 [/mm] + \ [mm] 5x_2 \le [/mm] 480$
$20 [mm] x_1 [/mm] +  [mm] 15x_2 \le [/mm] 480$

Dann habe ich überlegt, wie man modelliert, dass "keine Maschine mehr als 30 Minuten pro Tag länger läuft als jede der anderen". Dazu nehme ich mir 6 Variablen [mm] $y_0,\ldots,y_5$ (y_0 [/mm] für Bohrmaschine, rest für Fräsmaschinen) für die gelten muss

[mm] $y_i \le y_j [/mm] + 30$   für alle [mm] $i\not= [/mm] j$

Dann muss ich die Variablen noch irgendwie in Beziehung miteinander setzen und wie die Zielfunktion aussieht weiss ich auch noch nicht.

Naja bis hierhin erstmal. Ich werde noch weiter an der Aufgabe arbeiten. Für hilfreiche Ideen wäre ich dankbar.


LG
triad

        
Bezug
Werkstatt LP: Antwort
Status: (Antwort) fertig Status 
Datum: 20:26 Do 14.05.2015
Autor: meili

Hallo triad,

> Eine Werkstatt hat eine Bohr- und 5 Fräsmaschinen, um
> Teile zu produzieren, die aus 2 Werkstücken bestehen. Die
> Werkstücke müssen jeweils gebohrt und gefräst werden.
> Die Bearbeitungszeiten der Werkstücke sind:
>  
> Werkstück   Bohren (minuten)   Fräsen (minuten)
>  1           3                  20
>  2           5                  15
>  
> Alle Maschinen sollen so ausgelastet sein, dass keine
> Maschine mehr als 30 Minuten pro Tag länger läuft als
> jede der anderen. Die Fräsmaschinen seien dabei alle (so)
> gleichmäig (wie möglich) ausgelastet. Ein Arbeitstag hat
> 8 Stunden.
>  
> Formulieren Sie das Problem, die Maschinen so auszulasten,
> dass die maximale Anzahl von kompletten
>  Teilen produziert wird, als LP (Lineares Programm).
>  Hallo Leute,
>  
> mein Problem bei dieser Aufgabe besteht schon darin, welche
> und wie viele Variablen für was einzusetzen sind. Brauche
> ich nun für jede Maschine eine Variable oder für jedes
> Werkstück eine oder beides?

Beides, [mm] $x_1$ [/mm] und [mm] $x_2$ [/mm] für Anzahl der Werkstücke 1 und 2.
[mm] $y_0, y_1, \ldots y_5$ [/mm] Anzahl der Betriebsminuten für jede Maschine an einem Tag.
Wenn sich später herausstellt, dass eine Variable durch andere dargestellt
werden kann, ist das auch nicht schlimm.

>  
> Ich habe mal so angefangen: Die Summe der Bohr- und
> Fräsminuten überschreiten nicht 8 Stunden (480 Minuten),
> daher die zwei Nebenbedingungen [mm](x_i[/mm] für Werkstück i)
>  
> [mm]\ 3x_1 + \ 5x_2 \le 480[/mm]

[ok]

>   [mm]20 x_1 + 15x_2 \le 480[/mm]

Da es 5 Fräsmaschinen gibt, müsste diese Ungleichung
$20 [mm] x_1 [/mm] +15 [mm] x_2 \le [/mm] 5*480$
sein.

>  
> Dann habe ich überlegt, wie man modelliert, dass "keine
> Maschine mehr als 30 Minuten pro Tag länger läuft als
> jede der anderen". Dazu nehme ich mir 6 Variablen
> [mm]y_0,\ldots,y_5[/mm] [mm](y_0[/mm] für Bohrmaschine, rest für
> Fräsmaschinen) für die gelten muss
>  
> [mm]y_i \le y_j + 30[/mm]   für alle [mm]i\not= j[/mm]

Das ist erst die Hälfte der Bedingung.
Du hast die Laufzeit nach oben begrenzt,
aber sie muss auch noch nach unten eingegrenzt werden.


>  
> Dann muss ich die Variablen noch irgendwie in Beziehung
> miteinander setzen und wie die Zielfunktion aussieht weiss
> ich auch noch nicht.

Zielfunktion ist recht einfach: $max [mm] \{x_1+x_2\}$ [/mm]

Da ein komplettes Bauteil aus einem Werkstück 1 und einem Werkstück 2
besteht, muss [mm] $x_1 [/mm] = [mm] x_2$ [/mm] gelten.

>  
> Naja bis hierhin erstmal. Ich werde noch weiter an der
> Aufgabe arbeiten. Für hilfreiche Ideen wäre ich dankbar.
>  
>
> LG
>  triad

Gruß
meili

Bezug
                
Bezug
Werkstatt LP: Frage (beantwortet)
Status: (Frage) beantwortet Status 
Datum: 17:02 Fr 15.05.2015
Autor: triad

hallo und danke für deine antwort!


> Hallo triad,
>  
> > Eine Werkstatt hat eine Bohr- und 5 Fräsmaschinen, um
> > Teile zu produzieren, die aus 2 Werkstücken bestehen. Die
> > Werkstücke müssen jeweils gebohrt und gefräst werden.
> > Die Bearbeitungszeiten der Werkstücke sind:
>  >  
> > Werkstück   Bohren (minuten)   Fräsen (minuten)
>  >  1           3                  20
>  >  2           5                  15
>  >  
> > Alle Maschinen sollen so ausgelastet sein, dass keine
> > Maschine mehr als 30 Minuten pro Tag länger läuft als
> > jede der anderen. Die Fräsmaschinen seien dabei alle (so)
> > gleichmäig (wie möglich) ausgelastet. Ein Arbeitstag hat
> > 8 Stunden.
>  >  
> > Formulieren Sie das Problem, die Maschinen so auszulasten,
> > dass die maximale Anzahl von kompletten
>  >  Teilen produziert wird, als LP (Lineares Programm).
>  >  Hallo Leute,
>  >  
> > mein Problem bei dieser Aufgabe besteht schon darin, welche
> > und wie viele Variablen für was einzusetzen sind. Brauche
> > ich nun für jede Maschine eine Variable oder für jedes
> > Werkstück eine oder beides?
>  Beides, [mm]x_1[/mm] und [mm]x_2[/mm] für Anzahl der Werkstücke 1 und 2.
>  [mm]y_0, y_1, \ldots y_5[/mm] Anzahl der Betriebsminuten für jede
> Maschine an einem Tag.
>  Wenn sich später herausstellt, dass eine Variable durch
> andere dargestellt
> werden kann, ist das auch nicht schlimm.
>  
> >  

> > Ich habe mal so angefangen: Die Summe der Bohr- und
> > Fräsminuten überschreiten nicht 8 Stunden (480 Minuten),
> > daher die zwei Nebenbedingungen [mm](x_i[/mm] für Werkstück i)
>  >  
> > [mm]\ 3x_1 + \ 5x_2 \le 480[/mm]
>  [ok]
>  
> >   [mm]20 x_1 + 15x_2 \le 480[/mm]

>  Da es 5 Fräsmaschinen gibt,
> müsste diese Ungleichung
>  [mm]20 x_1 +15 x_2 \le 5*480[/mm]
>  sein.

o.k.

>  
> >  

> > Dann habe ich überlegt, wie man modelliert, dass "keine
> > Maschine mehr als 30 Minuten pro Tag länger läuft als
> > jede der anderen". Dazu nehme ich mir 6 Variablen
> > [mm]y_0,\ldots,y_5[/mm] [mm](y_0[/mm] für Bohrmaschine, rest für
> > Fräsmaschinen) für die gelten muss
>  >  
> > [mm]y_i \le y_j + 30[/mm]   für alle [mm]i\not= j[/mm]
>  Das ist erst die
> Hälfte der Bedingung.
>  Du hast die Laufzeit nach oben begrenzt,
> aber sie muss auch noch nach unten eingegrenzt werden.

Meinst du damit die nicht-negativitäts Bedingungen [mm] $y_i \ge [/mm] 0$ oder weitere Bedingungen der obigen Form? Denn die Aufgabenstellung verlangt doch nur "länger laufen", also nach oben beschränkt.

>  
>
> >  

> > Dann muss ich die Variablen noch irgendwie in Beziehung
> > miteinander setzen und wie die Zielfunktion aussieht weiss
> > ich auch noch nicht.
>  Zielfunktion ist recht einfach: [mm]max \{x_1+x_2\}[/mm]
>  
> Da ein komplettes Bauteil aus einem Werkstück 1 und einem
> Werkstück 2
>  besteht, muss [mm]x_1 = x_2[/mm] gelten.

Die Bedingung [mm] $x_1 [/mm] = [mm] x_2$ [/mm] ist dann einfach eine Nebenbedingung in meinem System?

Das LP sähe dann so aus:

[mm] \begin{matrix} \max & x_1+x_2 & & & \\ s.t. & 3x_1 + 5x_2 & \le & 480 & \\ & 20x_1+15x_2 & \le & 5\cdot{}480 & \\ & y_i & \le & y_j +30 & \forall i\not= j,\ i,j\in\{0,\ldots,5\} \\ & \glqq\mbox{2. Hälfte der Bedingung}\grqq & & & \\ & x_1 & = & x_2 & \\ & x_i & \ge & 0 & i=1,2 \\ & y_i & \ge & 0 & i=0,\ldots,5 \\ \end{matrix} [/mm]

Wie setze ich jetzt noch die [mm] $x_i$ [/mm] mit den [mm] $y_i$ [/mm] in Beziehung? Das muss ich
offensichtlich tun, sonst könnte ich die Bedingungen mit [mm] $y_i$ [/mm] im LP einfach weglassen.
Wenn man nach dem Prinzip "3 Minuten bohren und 20 Minuten fräsen ergibt ein Werkstück 1" geht,
wären das die Nebenbedingungen

[mm] $3y_0 [/mm] + [mm] 20y_i [/mm] = [mm] x_1 \qquad i=1,\ldots,5$ [/mm]

Analog für Werkstück 2. Aber stimmt das?

LG
triad

Bezug
                        
Bezug
Werkstatt LP: Antwort
Status: (Antwort) fertig Status 
Datum: 09:44 Sa 16.05.2015
Autor: meili

Hallo triad,

> hallo und danke für deine antwort!
>  
>
> > Hallo triad,
>  >  
> > > Eine Werkstatt hat eine Bohr- und 5 Fräsmaschinen, um
> > > Teile zu produzieren, die aus 2 Werkstücken bestehen. Die
> > > Werkstücke müssen jeweils gebohrt und gefräst werden.
> > > Die Bearbeitungszeiten der Werkstücke sind:
>  >  >  
> > > Werkstück   Bohren (minuten)   Fräsen (minuten)
>  >  >  1           3                  20
>  >  >  2           5                  15
>  >  >  
> > > Alle Maschinen sollen so ausgelastet sein, dass keine
> > > Maschine mehr als 30 Minuten pro Tag länger läuft als
> > > jede der anderen. Die Fräsmaschinen seien dabei alle (so)
> > > gleichmäig (wie möglich) ausgelastet. Ein Arbeitstag hat
> > > 8 Stunden.
>  >  >  
> > > Formulieren Sie das Problem, die Maschinen so auszulasten,
> > > dass die maximale Anzahl von kompletten
>  >  >  Teilen produziert wird, als LP (Lineares Programm).
>  >  >  Hallo Leute,
>  >  >  
> > > mein Problem bei dieser Aufgabe besteht schon darin, welche
> > > und wie viele Variablen für was einzusetzen sind. Brauche
> > > ich nun für jede Maschine eine Variable oder für jedes
> > > Werkstück eine oder beides?
>  >  Beides, [mm]x_1[/mm] und [mm]x_2[/mm] für Anzahl der Werkstücke 1 und
> 2.
>  >  [mm]y_0, y_1, \ldots y_5[/mm] Anzahl der Betriebsminuten für
> jede
> > Maschine an einem Tag.
>  >  Wenn sich später herausstellt, dass eine Variable
> durch
> > andere dargestellt
> > werden kann, ist das auch nicht schlimm.
>  >  
> > >  

> > > Ich habe mal so angefangen: Die Summe der Bohr- und
> > > Fräsminuten überschreiten nicht 8 Stunden (480 Minuten),
> > > daher die zwei Nebenbedingungen [mm](x_i[/mm] für Werkstück i)
>  >  >  
> > > [mm]\ 3x_1 + \ 5x_2 \le 480[/mm]
>  >  [ok]
>  >  
> > >   [mm]20 x_1 + 15x_2 \le 480[/mm]

>  >  Da es 5 Fräsmaschinen
> gibt,
> > müsste diese Ungleichung
>  >  [mm]20 x_1 +15 x_2 \le 5*480[/mm]
>  >  sein.
>  
> o.k.
>  
> >  

> > >  

> > > Dann habe ich überlegt, wie man modelliert, dass "keine
> > > Maschine mehr als 30 Minuten pro Tag länger läuft als
> > > jede der anderen". Dazu nehme ich mir 6 Variablen
> > > [mm]y_0,\ldots,y_5[/mm] [mm](y_0[/mm] für Bohrmaschine, rest für
> > > Fräsmaschinen) für die gelten muss
>  >  >  
> > > [mm]y_i \le y_j + 30[/mm]   für alle [mm]i\not= j[/mm]
>  >  Das ist erst
> die
> > Hälfte der Bedingung.
>  >  Du hast die Laufzeit nach oben begrenzt,
> > aber sie muss auch noch nach unten eingegrenzt werden.
>  
> Meinst du damit die nicht-negativitäts Bedingungen [mm]y_i \ge 0[/mm]
> oder weitere Bedingungen der obigen Form? Denn die
> Aufgabenstellung verlangt doch nur "länger laufen", also
> nach oben beschränkt.

Ja, $ [mm] y_i \ge [/mm] 0$ kannst du zur Klarstellung mit aufnehmen.
Das habe ich aber nicht gemeint, z.B. dürfte Fräsmachine 1 nicht
[mm] $y_1 [/mm] = 180$ Minuten laufen und Fräsmaschine 4 [mm] $y_4 [/mm] = 360$ Minuten.
Dazu habe ich aber zu sehr die einzelne Gleichung angesehen, und nicht
bedacht, dass wenn man alle Gleichungen
[mm] $y_i \le y_j [/mm] + 30, i [mm] \not= [/mm] j, [mm] \mbox{ für alle } [/mm] i,j [mm] \in \{0,1,2,3,4,5\}$ [/mm] zusammen nimmt, dies gewährleitet ist.

>  
> >  

> >
> > >  

> > > Dann muss ich die Variablen noch irgendwie in Beziehung
> > > miteinander setzen und wie die Zielfunktion aussieht weiss
> > > ich auch noch nicht.
>  >  Zielfunktion ist recht einfach: [mm]max \{x_1+x_2\}[/mm]
>  >  
> > Da ein komplettes Bauteil aus einem Werkstück 1 und einem
> > Werkstück 2
>  >  besteht, muss [mm]x_1 = x_2[/mm] gelten.
>  
> Die Bedingung [mm]x_1 = x_2[/mm] ist dann einfach eine
> Nebenbedingung in meinem System?

Ja.

>  
> Das LP sähe dann so aus:
>  
> [mm]\begin{matrix} \max & x_1+x_2 & & & \\ s.t. & 3x_1 + 5x_2 & \le & 480 & \\ & 20x_1+15x_2 & \le & 5\cdot{}480 & \\ & y_i & \le & y_j +30 & \forall i\not= j,\ i,j\in\{0,\ldots,5\} \\ & \glqq\mbox{2. Hälfte der Bedingung}\grqq & & & \\ & x_1 & = & x_2 & \\ & x_i & \ge & 0 & i=1,2 \\ & y_i & \ge & 0 & i=0,\ldots,5 \\ \end{matrix}[/mm]
>  
> Wie setze ich jetzt noch die [mm]x_i[/mm] mit den [mm]y_i[/mm] in Beziehung?
> Das muss ich
>  offensichtlich tun, sonst könnte ich die Bedingungen mit
> [mm]y_i[/mm] im LP einfach weglassen.
>  Wenn man nach dem Prinzip "3 Minuten bohren und 20 Minuten
> fräsen ergibt ein Werkstück 1" geht,
>  wären das die Nebenbedingungen
>  
> [mm]3y_0 + 20y_i = x_1 \qquad i=1,\ldots,5[/mm]

Ja, wobei das mit den [mm] $y_i$ [/mm] noch etwas ungenau ist.

Besser:

[mm] $3x_1 +5x_2 [/mm] = [mm] y_0$ [/mm]

[mm] $20x_1 [/mm] + [mm] 15x_2 [/mm] = [mm] \summe_{i=1}^{5} y_i$ [/mm]

>  
> Analog für Werkstück 2. Aber stimmt das?

Ausserdem würde ich noch aufnehmen: [mm] $x_1, x_2 \in \IN$, [/mm]
denn es sollen nur vollständige Teile produziert werden.

>  
> LG
>  triad

Gruß
meili

Bezug
Ansicht: [ geschachtelt ] | ^ Forum "Diskrete Optimierung"  | ^^ Alle Foren  | ^ Forenbaum  | Materialien


^ Seitenanfang ^
www.matheraum.de
[ Startseite | Forum | Wissen | Kurse | Mitglieder | Team | Impressum ]